Xem bài viết đơn
Old 25-12-2010, 09:05 PM   #6
toanlc_gift
+Thành Viên+
 
toanlc_gift's Avatar
 
Tham gia ngày: Jan 2009
Đến từ: FU
Bài gởi: 171
Thanks: 31
Thanked 142 Times in 80 Posts
Trích:
Nguyên văn bởi jakelong View Post
Các pác Cm giùm em bài này với cách sử dụng Bất Đẳng Thức AM-GM Cho 2 Số Nhé
Cho $x,y,z > 0 $ và $xyz=1 $ Cm:
$(1+\frac{x}{y})(1+\frac{y}{z})(1+\frac{z}{x})\geq (1+x)(1+y)(1+z) $
quên mất còn giả thiết Bất Đẳng Thức AM-GM Cho 2 Số Nhé nữa ^^!
$(x + y)(x + z) = ({x^2} + yz) + x(y + z) \ge 2\sqrt {x({x^2} + yz)(y + z)} $
$= 2\sqrt {({x^3} + 1)(y + z)} = 2\sqrt {(x + 1)(\frac{{{{(x + 1)}^2}}}{4} + \frac{{3{{(x - 1)}^2}}}{4})(y + z)} $
$\ge \sqrt {{{(x + 1)}^3}(y + z)} $
làm 2 bđt tương tự rồi nhân lại và rút gọn ta đc đpcm
[RIGHT][I][B]Nguồn: MathScope.ORG[/B][/I][/RIGHT]
 
toanlc_gift is offline   Trả Lời Với Trích Dẫn
The Following 2 Users Say Thank You to toanlc_gift For This Useful Post:
jakelong (25-12-2010), wikipedia1995 (25-12-2010)
 
[page compression: 8.47 k/9.56 k (11.47%)]